The Most Intimidating Integral I've Ever Seen

Поділитися
Вставка
  • Опубліковано 11 вер 2024
  • This Putnam Series was given on the Putnam exam in 1997 (Problem A3). Let's figure out a way to deal with this Putnam problem!
    For those that don't know, the Putnam math competition features some super interesting and challenging problems!
    🙏Support me by becoming a channel member!
    / @brithemathguy
    Disclaimer: This video is for entertainment purposes only and should not be considered academic. Though all information is provided in good faith, no warranty of any kind, expressed or implied, is made with regards to the accuracy, validity, reliability, consistency, adequacy, or completeness of this information.
    Putnam 1997 A3
    Putnam Exam 1997 A3
    #math #brithemathguy #putnam

КОМЕНТАРІ • 262